- Wed Jun 01, 2016 4:20 pm
#25925
Complete Question Explanation
(The complete setup for this game can be found here: lsat/viewtopic.php?t=5457)
The correct answer choice is (A)
The question stem establishes that S lectures second and Z lectures fourth. Since T lectures before S (second rule), we can conclude that T lectures first. Likewise, V lectures before Z (third rule), forcing V into the third position:
Among the remaining three variables, P lectures before W (first rule), whereas Y does not lecture seventh (fifth rule). Therefore, W must lecture seventh: The only uncertainty in the game, then, is the order in which P and Y deliver their lectures. Thus, in a Could Be True question such as this one, you should immediately seek answer choices that contain either P or Y. Only answer choices (A) and (E) qualify, and answer choice (A) is the only one that could be true.
(The complete setup for this game can be found here: lsat/viewtopic.php?t=5457)
The correct answer choice is (A)
The question stem establishes that S lectures second and Z lectures fourth. Since T lectures before S (second rule), we can conclude that T lectures first. Likewise, V lectures before Z (third rule), forcing V into the third position:
Among the remaining three variables, P lectures before W (first rule), whereas Y does not lecture seventh (fifth rule). Therefore, W must lecture seventh: The only uncertainty in the game, then, is the order in which P and Y deliver their lectures. Thus, in a Could Be True question such as this one, you should immediately seek answer choices that contain either P or Y. Only answer choices (A) and (E) qualify, and answer choice (A) is the only one that could be true.
You do not have the required permissions to view the files attached to this post.